What is the best unit to measure the capacity of a washing machine.

A) milliliter

B) liter

Answers

Answer 1
the best unit to measure the capacity of a washing machine is B) liter
Answer 2

Answer:

Liter

Step-by-step explanation:

Milliliters are used for much smaller amounts of liquid.


Related Questions

While training for a marathon, you consume at least 2200 calories a day. For one session of exercise, you consume 400 calories. How many calories do you consume for the rest of the day?

Answers

The amount of calories you consume for the rest of the day is 1800

How to determine the amount of calories you consume for the rest of the day

From the question, we have the following parameters that can be used in our computation:

Total amount of calories for a day = 2200 calories

Total amount of calories for one session = 400 calories

The above parameters can be represented as

Total amount of calories for one session + Total amount of calories for the end of the day = Total amount of calories for a day

Substitute the known values in the above equation, so, we have the following representation

400 + Total amount of calories for the end of the day = 2200

Evaluate the like terms

Total amount of calories for the end of the day = 2200 - 400

So, we have

Total amount of calories for the end of the day = 1800

Hence, the amount of calories is 1800

Read more about subtraction at

brainly.com/question/17301989

#SPJ1

Solve the inequality. Graph your solution. $$ - 5.3 x > 21 |

Answers

The solution to the inequality - 5.3 + x > 21 is x > 26.3

In mathematics, inequality refers to a relation which makes a non-equal comparison between two numbers or other mathematical expressions. In other words, it is a relationship between two expressions or values that are not equal to each other. It is used generally used to compare two numbers on the number line by their size.

In order to solve the given inequality - 5.3 + x > 21, add 5.3 both sides.

- 5.3 + x + 5.3 > 21 + 5.3

x > 26.3

The solution means that the value of x is greater than 26.3. The solution is graphed on a number line.

Learn more about Inequality:

https://brainly.com/question/25275758

#SPJ4

when testing individual coefficients of a multiple regression model, what is the sampling distribution?

Answers

When testing individual cofficients of a Multiple regression model, i.e y = β + β₁x₁ + β₂x₂ + ---+βₖxₖ + ε where β , β₁ , --- , βₖ are regression cofficients

the sampling distribution is tested by hypothesis testing.

Testing the Regression Coefficients:

An individual regression coefficient, if we want to test if there is a relationship between the dependent variable and the independent variable.

for the test and identify the significance level α.

The p-value is the sum of the area in the tails of the t -distribution. The t-score and degrees of freedom are t , df = n−k−1

Compare the p-value to the significance level and state the outcome of the test:

If p-value ≤ α, reject H₀ in favour of Ha .

The results of the sample data are significant. There is sufficient evidence to conclude that the null hypothesis H₀ is an incorrect belief and that the alternative hypothesis. Hₐ is most likely correct.

If p-value >α , do not reject H₀

The results of the sample data are not significant. There is not sufficient evidence to conclude that the alternative hypothesis Hₐ may be correct.

Write down a concluding sentence specific to the context of the question.

To learn more about Hypothesis testing t-statistic, refer :

https://brainly.com/question/4232174

#SPJ4

Triangle ABC is dilated. The image is A'B'C'

Find the value of x.

Answers

Answer:

x = 2

Step-by-step explanation:

6 : 4 = 3 : x

x = 4 * 3 / 6

x = 4/2

x = 2

The value of x in the given figure is 2

What is dilation?

Dilation is a transformation, which is used to resize the object. Dilation is used to make the objects larger or smaller. This transformation produces an image that is the same as the original shape. But there is a difference in the size of the shape.

Given that, Triangle ABC is dilated into A'B'C'

We know that dilated triangles are similar

Hence, their corresponding sides are in equal ratio,

BC : B'C' = AC : A'C'

6 : 4 = 3 : x

x = 4 × 3 / 6

x = 4/2

x = 2

Hence, The value of x in the given figure is 2

For more references on dilation, click;

https://brainly.com/question/13176891

#SPJ2

help please i need it

Answers

Answer:

since a || b

<A = <ABb = 60°

in a triangle the sum of all angles is 180°

so,

30 + 60 + x = 180

x= 180-90

x= 90

The length of the base edge of a pyramid with a regular hexagon base is represented as x. The height of the pyramid is 3 times longer than the base edge. The height of the pyramid can be represented as.

Answers

The height of the pyramid can be represented ,as per the given details as 3x.

The volume is 3/2 times 3 x 3, and the hexagon base's area is six times greater than the equilateral triangle's.

A pyramid's base edge's length is equal to x units.

In other words, the pyramid's height is three times as long as its base edge.

The pyramid's height is equal to 3x

A three-dimensional structure with a polygon at its base is referred to as a pyramid. You must be familiar with The Great Pyramid of Giza, which was built using a similar framework. This structure has a distinct shape because each corner is connected to a single apex.

To learn more about height

https://brainly.com/question/10726356

#SPJ4

40 times 3453 divided by 24=

Answers

Answer:

Step-by-step explanation:

57,555

Answer:5755

Step-by-step explanation: 40 X 3453 = 138,120

138,120 / 24 = 5755

Question
.Jeff and Lauryn are long-distance runners.

Jeff's race time, based on his average pace, is represented by the equation y=9x, where x is the number of miles and y is the number of minutes.

Lauryn's average pace is represented in this graph.


If Jeff and Lauryn run a 10k (a 6.2-mile race), who will win? Why?

Select two answers: one for the winner of the race and one for the reason they won.

Responses

Lauryn's pace is 9 minutes per mile, so Jeff is slower at 10 minutes per mile.
Lauryn's pace is 9 minutes per mile, so Jeff is slower at 10 minutes per mile.

Lauryn wins.
Lauryn wins.

Lauryn's pace is 10 miles per minute, so Jeff is slower at 9 miles per minute.
Lauryn's pace is 10 miles per minute, so Jeff is slower at 9 miles per minute.

Lauryn's pace is 10 minutes per mile, so Jeff is faste

Answers

Lauryn's race time equation is y = 10x. Thus,  Lauryn's pace is 10 minutes per mile, so Jeff is faster. Jeff will win.

How to determine who will win the race between Jeff and Lauryn?

Given that:

Jeff's race time is represented by the equation y=9x

You will find Lauryn's race time equation in the graph:

The equation of the line is of the form y = mx

where m is the slope

m = 20-0 / 2-0 = 20/2 = 10

Thus,  Lauryn's race time equation is y = 10x

For a 10k, the race time for Jeff will be:

y = 9x

y = 9×10 = 90 minutes

For a 10k, the race time for Lauryn will be:

y = 10x

y = 10×10 = 100 minutes

Therefore, Lauryn's pace is 10 minutes per mile, so Jeff is faster. Jeff wins.

Learn more about equation of a line on:

https://brainly.com/question/18831322

#SPJ1

The height (h) of an object after t seconds is given by the equation h=-2t²-9t+
56? In how many seconds will the object strike the ground?

Answers

Answer:

7/2 second

Step-by-step explanation:

The equation is: h= -2t²-9t+56

h = the height

We ask how many seconds the object will strike the ground. That means the h will be 0, and we have the equation!

0 = -2t^2 -9t + 56

First, we factor the equation and get

-(2t - 7) (t + 8) = 0

If any individual factor on the left side of the equation equals 0, the entire expression will be similar to 0. So we have the equations

2t - 7 = 0                    

t + 8 = 0

t = 7/2 and -8

Because a time can't be negative, so the answer is 7/2 second

So, the answer is in 7/2 seconds, the object will strike the ground.

.

a straight line passes through points (2,15) and (6,39) what is the y intercept?

Answers

Considering the expression of a line, the y intercept has a value of 3.

Linear equation

A linear equation o line can be expressed in the form y = mx + b

where

x and y are coordinates of a point.m is the slope.b is the ordinate to the origin and represents the coordinate of the point where the line crosses the y axis.

Knowing two points (x₁, y₁) and (x₂, y₂) of a line, the slope m of said line can be calculated using the following expression:

m= (y₂ - y₁)÷(x₂ - x₁)

Substituting the value of the slope m and the value of one of the points in the expression of a linear equation, the value of the ordinate to the origin b can be obtained.

y intercept in this case

Being (x₁, y₁)=(2,15) and (x₂, y₂)=(6,39), the slope m can be calculated as:

m= (39 - 15)÷(6 - 2)

m= 24÷ 4

m= 6

Considering point 1 and the slope m, you obtain:

15= 6×2 + b

15= 12 +b

15 - 12= b

3=b

Finally, the value of the y intercept is 3.

Learn more about the equation of a line having 2 points:

brainly.com/question/12851029

brainly.com/question/19496333

#SPJ1

A tile is in the shape of a rectangle.
It is 14 centimeters long and 3 centimeters wide.
What is its perimeter?

Answers

Answer: The perimeter is 34 cm

Step-by-step explanation:

14cm + 14cm = 28cm

3cm + 3cm = 6cm

28cm + 6cm = 34 cm

Answer:

34

Step-by-step explanation:

Perimeter is equal to :

Length + length + width + width

So theres two ways we can do this:

2(14) + 2(3)

OR

14 + 14 + 3 + 3

either way, solving will give us an answer of 34

Five times a number is 375. What is the number?

Answers

Answer:

75.

Step-by-step explanation:

We need to divide 375 by 5 since we are looking for 5 times a number. This will give us the answer, 75.

Answer:

75.

Step-by-step explanation:

We need to divide 375 by 5 since we are looking for 5 times a number. so the answer is 75

PLEASE HELP I NEED AN ANSWER!!!

Given the linear function f(x) = 2x + 3 using a table of values

Answers

Answer:

[tex]\left[\begin{array}{cc}x&y\\1&5\\2&7\\3&9\end{array}\right][/tex]

Step-by-step explanation:

Note that f(x) is typically used in replacement of y. So in reality, to make the question simple to understand, we will replace f(x) with y.

y = 2x + 3

Remember to follow PEMDAS. PEMDAS is the order of operations, and stands for:

Parenthesis

Exponents (& Roots)

Multiplication

Division

Addition

Subtraction

~

You are finding a table of value, which results in something similar to:

[tex]\left[\begin{array}{cc}x&y\\1&5\\2&7\\3&9\end{array}\right][/tex]

Proof 1:

Plug in 1 for x in the given equation:

y = 2x + 3

y = 2(1) + 3

y = 2 + 3

y = 5

Proof 2:

Plug in 2 for x in the given equation:

y = 2x + 3

y = 2(2) + 3

y = 4 + 3

y = 7

Proof 3:

Plug in 3 for x in the given equation:

y = 2(3) + 3

y = 6 + 3

y = 9

~

Do this for as much values as you need.

Learn more about creating a table of value, here:

https://brainly.com/question/8629807

an2-25art 2 10) Which fraction represents 72-7-20 eXP expressed in simplest form? 2) X-5 X-4 3) x+5 4+4 4) 25 X + 20

Answers

The given fraction (x^2-25)/(x^2-x-20) expressed in simplest form is (x+5)/(x+4). (Option C)

A fraction is in simplest form if the numerator and denominator have no common factors other than 1. In order to solve the given fraction, the numerator and denominator must be factorized, and the common factor will be canceled out.

Factoring x^2 – 25 using the difference of squares formula that states that a^2 – b^2 = (a + b)(a - b)

x^2 – 25 = x^2 – 5^2 = (x + 5)(x – 5)

Factoring x^2 – x – 20,

x^2 – x – 20 = x^2 + 4x – 5x – 20 = x(x + 4) -5(x + 4) = (x + 4)(x – 5)

Hence, factor (x – 5) is there in both numerator and denominator, it is canceled out. Hence the fraction in the simplest form is:

(x + 5)(x – 5)/ (x + 4)(x – 5) = (x + 5)/(x + 4)

Note: The question is incomplete.  The complete question probably is:  What fraction represents(x^2-25)/(x^2-x-20) expressed in simplest form. A) 5/4 B) (x-5)/(x-4) C) (x+5)/(x+4) D)25/(x+20)

Learn more about Simplest form:

https://brainly.com/question/17227519

#SPJ4

help!! please and thanks!

Answers

Answer: Congruent

Step-by-step explanation:

quizziz a random variable x has a mean of 5 and a std dev of .40. a random variable y has a mean of 8 and a std dev of .60. what is the mean of x y

Answers

The required mean as calculated from the given data is 13.

To calculate the mean x + y

mean of x + mean of y

= 5 + 8

=13

In addition to the mode and median, the mean is one of the measurements of central tendency. Simply put, the mean is the average of the values in the given set. It indicates that values in a particular data set are distributed equally.

The total values provided in a datasheet must be added, and the sum must be divided by the total number of values in order to determine the mean. When all of the values are organized in ascending order, the

Standard Deviation

The indicator of dispersion is the standard deviation. It denotes how widely apart the data values are from the mean value.

Formula: σ=∑i=1nxi-μ2n

NOTE :

Complete question:

A random variable X has a mean of 5 and a st. dev of 0.40. A random variable Y has a mean of 8 and a st. dev of 0.60. If both are independent, what is the mean of X + Y?

To learn more about mean

https://brainly.com/question/29397951

#SPJ4

Please help! More points!

Answers

Answer:

LOL

Step-by-step explanation:

4. Car dealer Lisa Kovach paid 82% of a car's options totaling $3,098. She paid 85% on a base price of $15,480.
The destination charge was $890. What is the dealer's cost?
a. $13,158.00
b. $16,588.36
c. $18,020.36
d. $19.001.20

Answers

Part (c) is the correct option i.e. The total dealer's cost is $16588.36.

What is Percentage ?

Percentage, which is a relative figure used to denote hundredths of any quantity. Since one percent (symbolised as 1%) is equal to one hundredth of something, 100 percent stands for everything, and 200 percent refers to twice the amount specified.

Given, Cost paid for car's options = 82 % $3,098 = $2540.36

           Cost paid for base price = 85 % $15,480 = $13158.

           Destination charge = $890

∴ The total dealer's cost will be :

 = Cost paid for car's options +  Cost paid for base price +  Destination charge

=  $2540.36 + $13158 + $890

= $16588.36.

To learn more about Percentage, visit the link given below:

https://brainly.com/question/24877689

#SPJ1

Rewrite by separating off the final termn+1
∑ ​m(m+1)
m=1

Answers

The rewritten summation is [tex]\sum_{m=1}^{n}m(m+1)+(n+1)(n+2)[/tex].

A recursive function is a function that calculates subsequent terms by repeating or using its own prior terms, creating a succession of terms. By definition, the recursive function can be used to write the summation by separating the final term of the summation and adding a final term to the summation.

Given, the function is [tex]\sum_{m=1}^{n+1}m(m+1)[/tex].

First, we have to separate the final term from the given summation. The final term is m = n+1 which is present above the summation. For this replace the n+1 with n.

Then, add the term m(m+1) with m in the given summation replaced by n+1.

The new summation is rewritten as,

[tex]\begin{aligned}\sum_{m=1}^{n+1}&=\sum_{m=1}^{n}m(m+1)+(n+1)((n+1)+1)\\&=\sum_{m=1}^{n}m(m+1)+(n+1)(n+2)\end{aligned}[/tex]

The new summation is rewritten.

The complete question is -

Rewrite by separating off the final term [tex]\sum_{m=1}^{n+1}m(m+1)[/tex]

To know more about summation:

https://brainly.com/question/29334900

#SPJ4

there are 30 children in a class and they all have at least one cat or dog. 14 children have a cat, 19 children have a dog. what is the probability that a child chosen at random from the class has both a cat and a dog?

Answers

The probability that a child chosen at random from the class has both a cat and a dog is 1/10.

Suppose b is the total number of kids that have both:

• children having a cat Only must be 14 - b

• children having a dog Only must be 19 - b

We also get:

We also know there are 30 kids, therefore

⇒ (14 - b) + b + (19 - b) = 30

⇒ 33 - b = 30

b = 3

Consequently, we now understand:

3/30 = 1/10

Know more about probability at:

https://brainly.com/question/24756209

#SPJ4

Solve |2 +1| = 10x

{-9/2, 9/2}
{-11/2, 9/2}
{9/2, 11/2}

Answers

The solution to the equation |2 +1| = 10x is x = 3/10

How to determine the solution to the equation

From the question, we have the following parameters that can be used in our computation:

|2 +1| = 10x

Evaluate the sum in the above equation

So, we have the following representation

|3| = 10x

Remove the absolute bracket in the above equation

So, we have the following representation

3 = 10x

Divide both sides by 10

This gives

3/10 = x

Rewrite as

x = 3/10

Hence, the solution is 3/10

Read more about equation at

https://brainly.com/question/2972832

#SPJ1

Use substitution to determine which system is represented by the graph.
ty
(
20
-28 0
-20
Arc 29
20
X

Answers

The system of equations plotted on the graph are -

y = (-3/5)x + 9

y = - (1/2)x - 12

What is the system of equations?

In mathematics, a set of simultaneous equations, also known as a system of equations or an equation system, is a finite set of equations for which common solutions are sought

Given are two graphs as shown in the image.

The equation of line [1] can be written as -

y = {(9 - 0)/(0 - 15)}x + 9

y = (-9/15)x + 9

y = (-3/5)x + 9

The equation of line [2] can be written as -

y = {(12 - 0)/(0 + 24)}x - 12

y = - (1/2)x - 12

Therefore, the given system of equations is -

y = (-3/5)x + 9

y = - (1/2)x - 12

To solve more questions on system of equations, we get -

https://brainly.com/question/29775795

#SPJ1

Consider the parent function f (x) below.

Answers

Answer:

  see the attachment

Step-by-step explanation:

You want graphs of f(x -5) and f(x)+2 and their asymptotes, given exponential function f(x).

f(x-5)

The subtraction of 5 from the argument x causes the graph to be shifted to the right 5 units. The horizontal asymptote at y=0 is unchanged.

f(x)+2

The addition of 2 to every function value causes the graph to be shifted up 2 units. The horizontal asymptote is likewise shifted up 2 units to y=2.

The red and blue graphs are attached. The asymptotes are dashed lines.

Question 1 (Multiple Choice Worth 2 points)
(Evaluating Algebraic Expressions LC)
If w = 24, what is the value of 2w - 18?

Answers

Answer:

30

Step-by-step explanation:

2 multiplied by 24 = 48

48 - 18 = 30

translate the description as an algebraic expression: half the difference of 2 and b

Answers

The answer would be 1/2(2-b) hope this helps.

how many different ways are there for 10 women and 6 men to stand in a line so that no two men stand next to each other? hint: first position the women and then consider possible positions for the men.

Answers

As per the combination method, the number of ways for 10 women and 6 men to stand in the line is 1,207,084,032,000

Combination method:

In statistics, combination method refers the method of selection of r items from a set of n items where the order of selection does not matter

Given,

Here we need to find how many different ways are there for 10 women and 6 men to stand in a line so that no two men stand next to each other.

Here we know that, first consider the positions of the women, then the positions of the men

Then the possible ways are there to arrange ten women in a row is

=> 10!=3628800.

For the given condition is no two men stand next to each other.

Therefore, we need to find how many ways we can arrange 6 men in the 11 possible places

=> 11P6 = 11×10×9×8×7×6 = 332640.

Therefore, the resulting number is

=> 3628800 × 332640 = 1,207,084,032,000

To know more about Combination method here.

https://brainly.com/question/28998705

#SPJ4

Find the value of x in the triangle shown below.
8
3

Answers

Answer:

The answer should be 8.

Step-by-step explanation:

A small business earns a profit of $6500 in January and $17,500 in May. What is the rate of change in profit for this time period. Explain how you got it.

Answers

The rate of change in the profit for the five months earned by the given small business is $2750.

How to calculate the rate of change?

The rate of change explains the relationship between two quantities.

Consider two quantities such profit and time in a small business.

Then, the rate of change in the profit is calculated by the ratio of change in the profit for the particular period to the time. I.e.,

rate of change in the profit = (change in the profit)/time

or

r = (p2 - p1)/(t2 - t1)

Where p1 is the profit gained at time t1 and p2 is the profit gained at time t2.

Calculation:

Given that, a small business earns a profit of $6500 in January (the first month of the year) and $17,500 in May (the fifth month of the year).

So, here p1 = $6500; p2 = $17,500; t1 = 1; and t2 = 5

Then, the rate of change in the profit for this period is

r = (17,500 - 6500)/(5 - 1)

 = 11000/4

 = 2750

Therefore, the rate of change in the profit for the five months is $2750.

Learn more about the rate of change at the following link:

https://brainly.com/question/24313700

#SPJ1

a) a research article described a simple random sample of 584 smokers who tried to quit smoking by using either a nicotine patch or e-cigarettes, fake cigarettes producing nicotine in water vapor form. after six months, 7.3% of those using e-cigarettes had stopped smoking, compared with 5.8% of those wearing the patch. to see if quitting result is significantly associated with the method used, which statistical inference procedure should we use? matched

Answers

To see if quitting result is significantly associated with the method used, which statistical inference procedure which should be used is the Chi-square test for two-way table.

A random sample of 584 smokers who attempted to quit smoking and were observed using the method they used to help themselves to achieve their goal was taken in order to determine whether there is a correlation between the percentage of people who successfully stop smoking and the method they used to do so (nicotine-patch or e-cigarettes). The percentage of subjects who successfully quit smoking after six months.

The t-test is not appropriate in this situation because the population mean is being tested by this statistic.

The percentage of participants who successfully quit smoking using either technique is the study's dependent variable.

The variable must be categorizable in order to do Chi-Square tests, and since the data can be arranged in a 2x2 table, all anticipated frequencies must be greater than 5.

The Goodness-to-Fit test seeks to determine if a population adheres to a particular theoretical model.

In this case, it's important to compare the percentage of smokers who were able to successfully stop using both methods and the percentage who were unsuccessful.

To learn more about Chi-Square tests

https://brainly.com/question/14082240

#SPJ4

This graph shows quadrilateral ABCD

Quadrilateral A'B'C'D' is the image of quadrilateral ABCD after the following sequence of transformations:
reflection across the y-axis
translation 9 units to the left and 5 units up
Write the coordinates pf the vertices of quadrilateral A'B'C'D'.

Answers

The final coordinates of the vertices of quadrilateral A'B'C'D';

A''(12, 9)

B''(16, 2)

C''(12, -2)

D''(13, 2)

How to interpret the sequence of transformations?

The coordinates of the quadrilateral ABCD are;

A(-3, 4)

B(-7, -3)

C(-3, -7)

D(-4, -3)

Now, the first sequence of transformation is reflection across the y-axis. Now, the transformation rule for reflection across the y-axis is;

(x, y) → (−x, y) .

Thus, we now have;

A'(3, 4)

B'(7, -3)

C'(3, -7)

D'(4, -3)

It is now translated 9 units left and as such the transformation rule is;

((x + 9), y)

We now have;

A'(12, 4)

B'(16, -3)

C'(12, -7)

D'(13, -3)

It is now translated 5 units up and as such, the final coordinates are;

A''(12, 9)

B''(16, 2)

C''(12, -2)

D''(13, 2)

Read more about sequence of transformations at; https://brainly.com/question/4289712

#SPJ1

Other Questions
Which metaphors describe Robin Hood? Select two options. a sly fox brave and strong hoping to succeed a doorway to freedom careful to move in silence hiding himself from view.mark!!!!!!!!!!!!!!!!!!!!!!!!!!!!!!! Which led to the 1905 revolution the james family wants to save some money in interest by paying off their auto loan 12 months early. the 36-month loan required payments of $455 per month. the car cost $18,600 with a down payment of $6000. how much unearned interest did they save? In an experiment, the enzyme catecholase is mixed with various concentrations of its substrate, catechol. Each test tube begins with a certain catechol concentration, and all tubes contain the same concentration of enzyme. The rate of reaction (conversion of substrate to product) is measured for each of the samples, and the results are plotted on the graph below. Explain the shape of the curve in the graph What expenses do you need to budget for if you choose to buy a home? Check all that apply.A. a mortgage paymentB. a rent paymentC. homeowners insuranceD. renters insuranceE. property taxesF. a security depositG. a down paymentH. utility payments for each of the following molecules, would you expect greater solubility in water or in hexane? Which of the following runs on each node and ensures containers are running in a pod?O PodO EtcdO KubeletO Scheduler Where is Weed in "Of Mice and Men"? What trouble did George and Lennie have there? A person describes what an ambiguous image appears to be.Rorschach inkblot test What scenario can lead to pest infestation? Which of the following is true about Product Life Cycles?a) A Reminder promotion is normally used in the introductory phaseb) Profits peak in the maturity phasec) Early adopters buy in the introductory phased) Sales peak in the growth phasee) A informative promotion is most generally used the maturity phase How was the British attack at the Battle of Bunker Hill different from the one at the Battles of Lexington and Concord? At the Battle of Bunker Hill, the British attacked from ships and on foot. At the Battle of Bunker Hill, the British attacked with fewer soldiers. At the Battles of Lexington and Concord, the British arrived in canoes. At the Battles of Lexington and Concord, the British did not shoot their guns at all. though the director sets the pace for a production, the pace and rhythm of a production ultimately becomes the responsibility of its performers. State True or False your answer: a. Trueb. False Adults of any age who have coronary heart disease should be following a heart-healthy diet that limits all the following EXCEPT __________.- saturated and trans fats- cholesterol - protein from plant sources- sodium discuss the relationship between nick and jordan baker. how does it reflect, if at all, on the story of gatsby and daisy? a car manufacturer decides to promote the sales of his product by offering discounts online via anemail campaign. consumers simply have to click on a link in the email, and they will be able to avail the offer. theadvantage of using this technique is that it not only increases brand awareness for the company, but also: a piston presses down on a sealed cylinder filled with gas until the gas occupies a space that is half as big as before. as a result, the pressure on the gas is Which set of ordered pairs does not represent a function? a.{(3, 7), (1, 9), (5, 11)} b.{(9, 5), (4, 5), (1, 7)} c.{(2, 1), (3, 4), (2, 6)} Which of the following features of a 1H NMR spectrum give useful information about the structure of a compound? Choose all answers that apply.----------------------------------------The spin-spin splitting of signalsThe number of the signalsThe area under the signalsThe signal distance above the baselineThe chemical shift of the signals Le gustaria algo mas?